Lyle uses a streaming service to watch movies. Last month he paid $26.91 and watched 9 movies. If he watches 11 movies this month, how much will he pay the steaming service?

A. $32.89
B. $26.91
C. $2.99
D. 3.68

Answers

Answer 1

Answer:

A. $32.89

Step-by-step explanation:

Take the amount and divide by the number of movies watched

26.91 / 9 =2.99

Now multiply by the new number of movies he wants to watch, 11

2,99 *11

32.89

Answer 2

Answer:

A) $32.89

Step-by-step explanation:

We divided 26.91 by 9 to find how much it is per movie

26.91/9

2.99

It is 2.99 for 1 movie so now we multiply by 11 to find all 11 movies price

2.99x11

$32.89

Hopes this helps please mark brainliest if you would like to learn cross multiply way just let me know


Related Questions

If point A(-5, -7) lies on the terminal arm of an angle, determine the exact value for the
primary trigonometric ratios of the angle.

Answers

Answer:

[tex]sin(a)=\frac{y}{r}=-\frac{7\sqrt{74} }{74}[/tex]

[tex]cos(a)=\frac{x}{r} =-\frac{5\sqrt{74} }{74}[/tex]

[tex]tan(a)=\frac{y}{x} =\frac{7}{5}[/tex]

Step-by-step explanation:

Given the terminal side of an angle we can calculate the distance between the point given and the origin:

[tex]r=\sqrt{x^2+y^2}[/tex]

[tex]r=\sqrt{|(-5^2)+(-7^2)|}[/tex]

[tex]r=\sqrt{74}[/tex]

y = opposite side

x = adjacent side

r = hypotenuse

Now we have

[tex]r=\sqrt{74}[/tex]

[tex]x=-5[/tex]

[tex]y=-7[/tex]

[tex]sin(a)=\frac{y}{r}=-\frac{7\sqrt{74} }{74}[/tex]

[tex]cos(a)=\frac{x}{r} =-\frac{5\sqrt{74} }{74}[/tex]

[tex]tan(a)=\frac{y}{x} =\frac{7}{5}[/tex]

please help me dhdjjdjejejejdjejejejejejkenensndd

Answers

Answer: area=8cm

Step-by-step explanation:

Answer:

a) Area of shape = 8 square cm.

Step-by-step explanation:

Area of composite shape:

      Area of shape = area of rectangle + 2* area of triangle

Rectangle:

         l = 3 cm ; w = 2 cm

[tex]\sf \boxed{\text{Area of rectangle = l *w}}[/tex]

                             = 3 * 2

                             = 6 cm²

Triangle:

     b = 2 cm

     h = 1 cm

[tex]\sf \boxed{\text{Area of triangle=$\dfrac{1}{2}*b*h$}}[/tex]

                         [tex]\sf = \dfrac{1}{2}*2*1\\\\\\ = 1 \ cm^2[/tex]

Area of two triangles = 2 *1

                                    = 2 cm²

Area of shape = 6 + 2

                        = 8 cm²

b) Area of rectangle ABCD = 4 * 2

                                             = 8 cm²

what is the value of the expression y - x?​

Answers

Answer:

[tex]y-x=-10[/tex]

Step-by-step explanation:

In any trapezoid, the sum of all angles is 360°. Additionally, the sum of both angles on the same side of the trapezoid is 180°. Therefore, we know that:

[tex](7x+40)+(68-6y)=180[/tex]

[tex](17y+73)+(121-4x)=180[/tex]

To solve for [tex]x[/tex] and [tex]y[/tex], we must set up a system of equations and solve by substitution, elimination, or graphing.

To start, simplify the first formula, then isolate the [tex]x[/tex] variable:

[tex](7x+40)+(68-6y)=180[/tex]

[tex]7x-6y+68+40=180[/tex]

[tex]7x-6y+108=180[/tex]

[tex]7x-6y=72[/tex]

[tex]7x=72+6y[/tex]

[tex]x=\frac{72+6y}{7}=\frac{72}{7}+\frac{6}{7}y[/tex]

Next, simplify the second formula, then substitute the value above for [tex]x[/tex]:

[tex](17y+73)+(121-4x)=180[/tex]

[tex]-4x+17y+121+73=180[/tex]

[tex]-4x+17y+194=180[/tex]

[tex]-4(\frac{72}{7}+\frac{6}{7}y)+17y+194=180[/tex]  (substituted for [tex]x[/tex] here)

[tex]-\frac{288}{7}-\frac{24}{7}y+17y+194=180[/tex]

[tex]-\frac{288}{7}-\frac{24}{7}y+\frac{119}{7}y+\frac{1358}{7}=\frac{1260}{7}[/tex]

[tex]-288-24y+119y+1358=1260[/tex]

[tex]95y+1070=1260[/tex]

[tex]95y=190[/tex]

[tex]y=2[/tex]

Finally, substitute 2 for [tex]y[/tex] in the first formula as simplified:

[tex]7x=72+6y[/tex]

[tex]7x=72+6(2)[/tex]

[tex]7x=72+12[/tex]

[tex]7x=84[/tex]

[tex]x=12[/tex]

Therefore:

[tex]x=12[/tex]  and  [tex]y=2[/tex]

And the expression [tex]y-x[/tex] equals:

[tex]y-x=2-12=-10[/tex]

Estimate the sum or difference, by rounding each number to its largest place (front-end rounding)–36.673+39.999

Answers

Answer:

0

Explanation:

Given the difference:

[tex]-36.673+39.999[/tex]

In front-end rounding, we consider the number with the largest place value.

In -36.673, the number with the largest place value is 3.

The digit after 3 us 6, so we round up as follows:

[tex]-36.673\approx-40[/tex]

Likewise, In 39.999, the number with the largest place value is 3.

The digit after 3 us 9, so we round up as follows:

[tex]39.999\approx40[/tex]

Therefore:

[tex]-36.673+39.999\approx-40+40=0[/tex]

The diffrence is 0 using front-end rounding.

The table shows the rational approximation of several irrational numbers.
Which is the approximate value of √3?

Answers

The approximate value of √3 is 1.732 which is an irrational number.

Given that,

We have to find the value of root3.

√3 is an irrational number.

Real numbers that cannot be represented as a ratio are referred to as irrational numbers. Or to put it another way, irrational numbers are actual numbers that defy logic.

Irrational numbers are real numbers that cannot be represented by a straightforward fraction. These can't be stated as ratios, like p/q, where p and q are both integers, q≠0. In terms of statistics, it defies reason.

The approximate value of √3 is 1.732.

Therefore, The approximate value of √3 is 1.732 which is an irrational number.

To learn more about irrational visit: https://brainly.com/question/15837135

#SPJ9

Solve the following inequality, 6 < 3r-45 < 36 Which graph shows the correct solution?

Answers

Inequality solution

We know that:

6 < 3r-45 < 36

We want to re arrange the inequations:

Step 1: all terms with an unkown value on one side.

We want to let 3r "be alone" in the middle side:

6 < 3r-45 < 36

6 + 45 < 3r < 36 + 45 [adding 45 in each part]

51 < 3r < 81 [6 + 45 = 51 and 36 + 45 = 81]

Step 2: we want to clear r

51 < 3r < 81

51/3 < r < 81/3 [dividing by 3 all sides]

17 < r < 27 [51/3 = 17 and 81 /3 = 27]

Answer: 17 < r < 27GRAPH

Zeke wants to build a fence around his backyard with the 200 feet (ft) of fencing he bought at Lowe's. Zeke wants to use all of the fencing he bought. he also wants the length of the fencing to be 10 times the width. the equation A=200w-10w^2 can be used to find the area of the garden, where w is the width of the garden, in ft​

Answers

Considering the vertex of the quadratic equation, the width that will maximize the area of the garden, and the maximum area, are as follows:

Width: 10 ft.Maximum area: 1000 ft².

What is the vertex of a quadratic equation?

A quadratic equation is defined as follows:

y = ax² + bx + c.

The vertex can be either a maximum point or a minimum point, depending on the coefficient a, as follows:

Maximum: a < 0.Minimum: a > 0.

The coordinates of the vertex are given as follows:

x = -b/2a.y = -(b² - 4ac)/4a.

For this problem, the equation is:

A(w) = -10w² + 200w.

Hence the coefficients are:

a = -10, b = 200, c = 0.

The width that will maximize the area is:

w = -200/(2(-10)) = 200/20 = 10 ft.

The maximum area is of:

A = -(200)²/(4(-10)) = 1000 ft².

Missing Information

The problem is incomplete and could not be found on any search engine, hence we suppose that it asks for the width that will maximize the area of the garden, and the maximum area.

More can be learned about the vertex of the quadratic equation at https://brainly.com/question/24737967

#SPJ1

The radius of the front wheel of Terry's bike is 61cm.
Terry goes for a cycle and travels 60.32km.
How many full revolutions did Terry's front wheel complete?

Answers

In linear equation, 157 did Terry's front wheel complete.

What is a linear equation example?

Ax+By=C is the typical form for linear equations involving two variables. A linear equation in standard form is, for instance, 2x+3y=5.Finding both intercepts of an equation in this format is rather simple (x and y).

Terry goes for a cycle and travels 60.32km.

 so, 60.32 * 1000 ÷ ( 2 * 3.14 * 61 )

   = 60320 ÷ 2 * 3.14 * 61

   = 60320 ÷ 6.28  * 61

    = 60320 ÷ 383.08

     = 6032000/ 38308

     = 1508000/9577

    = 157

Learn more about linear equation

brainly.com/question/11897796

#SPJ1

I don’t under stand help

Answers

The answer is E

E is the same shape and around the same size as A. A tales up a total of around 6 squares as well as E. E would be a rotation of 90°

There are f flavors of ice cream at the shop. Dominic has sampled 7 of them. Choose the expression that shows the number of flavors Dominic has not sampled.

Answers

In linear equation, Jen planned to spent 3.75 hours paddleboarding.

What is a linear equation example?

Ax+By=C is the typical form for linear equations involving two variables. A linear equation in standard form is, for instance, 2x+3y=5.Finding both intercepts of an equation in this format is rather simple (x and y).

Dominic has sampled =  7

flavors of ice cream at shop = f

the expression that shows the number of flavors Dominic has not sampled  

                                        = 7 * f = 7f                

Learn more about linear equation

brainly.com/question/11897796

#SPJ1  

6(3n+10) whats the answer

Answers

Problem: 6(3n+10)

Solution:

Applying the distributive law of natural or real numbers, we obtain:

[tex]6(3n+10)\text{ = 6.3n+ 10.6 = }18n+60[/tex]

then, the correct answer is:

[tex]6(3n+10)\text{ =}18n+60[/tex]

three times the measure of an angle is equal to twice the measure of the angle's supplement. what is the measure of the angle

Answers

Answer:

The angle has measure 72°.

Step-by-step explanation:

Definition:

Two angles are supplementary angles if the sum of their measures equals 180°.

Let x and y be the supplementary angles.

According to the definition above, then,

x + y = 180

We can the above equation for y:

x + y = 180

y = 180 - x

The two angles have measures:

x

180 - x

Now we look carefully at this statement:

"three times the measure of an angle is equal to twice the measure of the angle's supplement"

We translate the above statement into an equation, using x for one angle, and 180 - x for the supplement.

3x = 2(180 - x)

Solve the equation for x.

Distribute 2 on the right side.

3x = 360 - 2x

Add 2x to both sides.

5x = 360

Divide both sides by 5.

x = 360/5

x = 72

The angle has measure 72°.

Three functions are shown.
Linear
Quadratic
Exponential
Which of the following statements is true?

Answers

The correct statement C. Exponential growth will always exceed linear growth.

What is termed as the Exponential growth?Exponential growth is a data pattern that exhibit good increases over time, resulting in the curve of such an exponential function.Whereas exponential growth is frequently used in corporate finance, the actuality is frequently more complex. In the case of a savings account, the implementation of exponential growth did work well because the rate of interest is assured and does not change over time. This isn't the case with the majority of investments.

For the given question;

In the end, a "exponential growth function" would always outperform a "linear growth function" because "x-values" tend to increase in continuation, and as such the change rate affiliated with the "exponential function" as well increases in continuation, whereas the "linear function's" change rate is considered constant.

Thus, Exponential growth will always exceed linear growth.

To know more about the Exponential growth, here

https://brainly.com/question/27161222

#SPJ1

The complete question is-

Three functions are shown.

A. Linear growth will always exceed exponential growth.

B. Quadratic growth will always exceed exponential growth.

C. Exponential growth will always exceed linear growth.

Which of the following determines the range of spectral lines produced during electron transition?

A.
the total number of energy levels the electron can jump to

B.
the lower energy level to which the electron returns

C.
the higher energy level to which the electron jumps

D.
the number of electrons under electron transition at the same time

Answers

The range of spectral lines produced during electron transition is determined by the total number of energy levels the electron can jump to.

Spectral lines are bright or dark lines over continuous spectrum which occur due to emission or absorption of energy.

When an electron jumps to or from one energy level to another energy level, spectral lines are produced. The range of spectral lines depends on the number of energy levels available to which the electron can jump. This depends the amount of energy gained/lost by the electron.

Thus, the range of spectral lines produced during electron transition is determined by the total number of energy levels the electron can jump to.

To learn more about the spectral lines refer here

https://brainly.com/question/16414977

#SPJ1

95% of ____ is 427.5

Answers

Answer: Hi! The answer is 406,125.

Step-by-step explanation: You can either go by 1%, meaning you divide 427,5 by 100 and then do x95, which gives you 95%, or you can do 427,5x0,95. Either way you get 406,125!

Answer:

406.125

Step-by-step explanation:

I don't understand this

Answers

Answer:

∠F = 29

Step-by-step explanation:

There is a property of angle of triangles.

Exterior angle = Sum of opposite interior angles

58 = x + x

58 = 2x

2x = 58

x = 58/2

x = 29

∠F = x = 29

many wholes are in 21 / 5

Answers

Divide the numbers:

21/5 = 4.2

Yvette maps out several locations in her town, with distances and angles between them. Two triangles are formed within the map.
To conclude that these triangles are congruent by SAS Congruence Postulate, what must the distance between the school and the park be

Answers

To be both triangles to be congruent the distance between the school and the park be 1.1 miles.

What is congruence?

If two figures are exactly the same in sense of their length side all things then they will be congruent.

If it is possible to superimpose one geometric figure on the other so that their entire surface coincides, that geometric figure is said to be congruent or to be in the relation of congruence.

As per the given two triangles,

The side 2.7 miles is the common side.

The angle is 52 degrees in both triangles same.

To be congruent any one side must be the same by the SAS rule.

Thus, the school-to-park corresponding side is school-to-store.

school to park = 1.1 miles

Hence "To be both triangles to be congruent the distance between the school and the park be 1.1 miles".

To learn more about congruence,

https://brainly.com/question/12413243

#SPJ1

Lines AB, CD, and LK intersect as shown in the figure below. AB I CDKA

Answers

The Alternate Exterior Angles Theorem states that, when two parallel lines are cut by a transversal , the resulting alternate exterior angles are congruent.

Given that:

[tex]\begin{gathered} m\angle\text{LRB}=86\degree,\text{ its alternate exterior angle is }m\angle\text{CSK} \\ \\ \text{Therefore,} \\ m\angle\text{CSK}=m\angle\text{LRB} \\ m\angle\text{CSK}=86\degree\text{ (final answer)} \end{gathered}[/tex]

16 oz harbor peanut butter for 2.49 or a 64 oz jar of peanut butter for 6.99 round everything up to the nearest cent or hundreth. yes mam

Answers

We have to find the "better buy" between two options:

1) 16 oz jar for $2.49

2) 64 oz jar for $6.99

We can compare this two options with the unit price of each one. The unit price will be expressed in "$ per oz" or "$/oz". The option with the smaller unit price is the "better buy".

We can calculate the unit price as teh quotient between the price and the weight of each option.

Unit price for Option 1:

[tex]u_1=\frac{2.49\text{ \$}}{16\text{ oz}}\approx0.16\frac{\$}{oz}_{}[/tex]

Unit price for Option 2:

[tex]u_2=\frac{6.99\text{ \$}}{64\text{ oz}}\approx0.11\frac{\$}{oz}[/tex]

As the Option 2 (64 oz jar) has a smaller unit price, the better buy is the 64 oz jar for $6.99.

Answer: the better buy is the 64 oz jar for $6.99.

Write the equation of the linear relationship in slope-intercept form, using decimals as needed.


x 25 35 45 55
y 85 77 69 61


The equation that represents this relationship is y =

Answers

The equation of the linear relationship in slope-intercept form is y = (-4/5)x + 105.

Given:

x 25 35 45 55

y 85 77 69 61

slope between (25,85) and (35,77) is m = 77-85/35-25

= -8/10

m= -4/5

y=mx+c passes though (25,85)

85 = -4/5*25 + c

85 = -4*5+c

85 + 20 = c

c = 105.

y = mx + c

y = (-4/5)x + 105

Therefore the equation of the linear relationship in slope-intercept form is y = (-4/5)x + 105.

Learn more about the slope intercept form here:

https://brainly.com/question/9682526

#SPJ1

Xavier Knox's annual salary is $61,100. He is paid semimonthly. His personal exemptions total $4,000. How
much does his employer deduct from each of Knox's paychecks for state income tax of 3.5 percent?
a. $166.54
c.
b. $83.27
$83.13
d. $166.26

Answers

The amount that Xavier Knox's employer deducts from his paychecks for state income tax is b. $83.27

How to find the state income tax?

First find his taxable income:

= Annual salary - Personal exemptions

= 61, 100 - 4, 000

= $57, 100

The state income tax per year is:

= 57, 100 x 3.5% state income tax rate

= $1, 998.50

The semimonthly state income tax deducted is:
= 1, 998.50 / (12 months x twice a month)

= $83.27

In conclusion, on a semimonthly basis, $83.27 is deducted for state income tax.

Find out more on state income tax at https://brainly.com/question/23687799

#SPJ1

A bag contains 15 cards numbered 1 through 15. A card is randomly chosen from the bag. What is the probability that the card has an odd number on it? Write your answer as a fraction in simplest form.

Answers

There are 15 cards numbers are from 1 to 15.

From the numbers 1 to 15:

• 8 numbers, are ,odd

,

• 7 numbers, are ,even,

The probability is the possibility of an event occuring.

Probability of event to happen P(A) = Number of favourable outcomes/Total Number of outcomes

Thus, the probability of an odd card is:

total number of odd cards/total number of cards

= 8/15

the Correct Answer is:

[tex]\frac{8}{15}[/tex]

Clarissa and Koko solve 3x + 5 = 2x + 4 by graphing the related function. Is either of them correct? Explain your reasoning.

Answers

Answer:

clarissa

Step-by-step explanation:

there are 55 stickers in each box how many stickers are in 7 boxes

Answers

answer: 385 stickers
explanation: 7x55=385

Nell participated in a 3-day charity walk. Sheraised $0.50 for each 1/3 of mile that she walked. Thefirst day, Nell walked 12 miles. The second day, shewalked 8 miles. The third day, she walked 16 miles.How much money did Nell raise?

Answers

the Numbers of days = 3

The second statement(She raised $0.50 for each 1/3 of mile that she walked) means

1/3 miles = $0.50

For first day walk we have:

1/3 miles = $0.50

12 miles = 12 x 0.50 x 3 = $18

For Second day walk we have:

1/3 miles = $0.50

8miles = 8 x 0.50 x 3 = $12

For Third day walk we have:

1/3 miles = $0.50

16miles = 16 x 0.50 x 3 = $24

The amount she raised for the three days becomes

$18 + $12 + $24

=$54

use the product, quotient, and power rules of logarithms to rewrite the equation as a single logarithm.

Answers

We are given the following expression:

[tex]\log a-3\log b+4\log c[/tex]

we are asked to simplify this expression. To do that we will first use the following property:

[tex]a\log b=\log ^{}b^a[/tex]

we will apply this to the second and third terms, like this:

[tex]\log a-\log b^3+\log c^4[/tex]

Now we will use the following property:

[tex]\log a-\log b=\log (\frac{a}{b})[/tex]

we will use this property for the first and second terms:

[tex]\log (\frac{a}{b^3})+\log c^4[/tex]

Now we will use the following property:

[tex]\log a+\log b=\log ab[/tex]

We will use the property in the last two terms, like this:

[tex]\log (\frac{ac^4}{b^3})[/tex]

And thus, we have simplified the logarithmic expression into one single logarithm

I need help please with [tex]6409 \div 61[/tex]. I know the answer is 105.06the problem is it has to be written as a a whole number with a fraction. we put 105 and 3/50 but its saying it's wrong.

Answers

Answer:105 4/61Explanation:

Given the expression

[tex]\frac{6409}{61}[/tex]

Next is to know how many 61 can go in 6409. Using the calculator, you can see that it is 105 with a remainder of 4

We will then have to express the fraction in the form:

[tex]Q\text{ +}\frac{R}{D}[/tex]

Q is the quotient = 105

R is the remainder = 4

D is the divisor = 61

Substituting these values into the expression we will have:

[tex]\begin{gathered} \frac{6409}{61}=105+\frac{4}{61} \\ \frac{6409}{61}=105\frac{4}{61} \end{gathered}[/tex]

Hence the solution written as a whole number and fraction is 105 4/61

Find the distance between the points J(-8, 0) and K(1, 4).

Answers

The distance between two points of coordinates (x₁, y₁ ) and (x₂, y₂) is calculated using the formula:

[tex]undefined[/tex]

Which of the following represents vector vector t equals vector PQ in trigonometric form, where P (–13, 11) and Q (–18, 2)?

Answers

SOLUTION

The coordinate of the vector P and Q are

[tex]\begin{gathered} P(-13,\text{ 11)} \\ \text{And } \\ Q(-18,2) \end{gathered}[/tex]

To find the vector PQ. we have

[tex]\begin{gathered} t=\bar{PQ} \\ PQ\text{ is having the coordinate } \\ PQ=(-18-(-13),2-11)=(-5,-9) \end{gathered}[/tex]

To find the vector, we use

[tex]\begin{gathered} r=\sqrt[]{x^2+y^2} \\ \text{Where } \\ x=-5,y=-9 \\ r=\sqrt[]{(-5)^2+(-9)^2}=\sqrt[]{25+81}=\sqrt[]{106}=10.296 \end{gathered}[/tex]

Then we obtain the angle using

[tex]\begin{gathered} \text{tan}\theta=(\frac{y}{x})_{} \\ \text{Substituting the value of x and y, we have } \\ \tan \theta=(\frac{9}{5})=\tan \theta=(1.8) \end{gathered}[/tex]

Hence

[tex]\begin{gathered} \tan \theta=1.8 \\ \theta=\tan ^{-1}(1.8) \\ \theta=60.945 \end{gathered}[/tex]

Hence

The vector in trigonometry form will be

[tex]\begin{gathered} t=r(i\cos \theta+j\sin \theta) \\ \text{Then} \\ t=10.296\cos 60.945i+10.296\sin 60.945j \end{gathered}[/tex]

Therefore

t= 10.296 cos 60.945 i + 10.296 sin 60.945j

Answer: Option C(third option ).

Other Questions
For each shape in the table list three examples of real world objects that could be modeled by the shape use your experiences, the Internet, newspapers, magazines, or other resources to uncover examples1.) rectangular prism2.) triangular prism3.) cylinder4.) cone5.) pyramid6.)sphere What is the incident known as that was launched by the nazis on november 9, 1938 that is considered the beginning of the holocaust?. How do you work the following problem:517 37/50 + 312 3/100? and I also need to turn in to a decimal Question 2 of 5Which sentence most clearly describes the medium of a story?OA. Mermaids seem out of place in a coming-of-age story like thisone.OB. The story suffers from not having any characters who arerelatable.OC. The novel gives more details about the character than the show.OD. Drawing out the climax really ramped up the tension in the story.SUBMIT Which of the following BEST Dont completes the above analogy? What is the radius and diameter of the following circle?18 cmRadius =cmDiameter =cm tenemos $5000 en una cuenta. A final de cada mes se ingresa un 5% del dinero que hay en la cuenta en dicho momento. Calcular el dinero que habr en la cuenta despus de un trimestre en que porcentaje ha subido la cantidad inicial? Determine the point on the graph of the unit circle that corresponds to pi. Then find cos pi and sin pi, and state which functions are undefined for pi. In the Harry Potter novels, we see Harry starting out as a scared orphan who develops into a powerful hero who seeks to fight evil by the last novel. This is an example of...*emotional appealantagonistcomplex/dynamicflat/static on4Suppose that the pressure of 1.23 L of gas is 300.6 mm Hg when the temperature is 218.5 K. At what temperature is the volume 8.32 L and the pressure 802.75 mm Hg?O a. 3950O b. 8610O c. 1250O d. 12.1Check How did the reporter state probability for rain last week compared to the actual results if a rectangle with an area of 42 units^2 has a length of seven units what is the rectangles primeter? Please help me with this timed practice problem.The first step have different options: substract, distribute, divide, add. I don't understand problem like this : h >4 Write the given function as the composition of two functions.y=15 + 6xChoose the correct answer below.OA. If f(x) = -x and g(x) = 15+ 6x, then y = f(g(x)].1B. If f(x)=x and g(x) = -OC. If f(x) =xCOD. If f(x) = -115+ 6x'then y = f[g(x)].and g(x)= 15+ 6x, then y = f[g(x)].and g(x) = 15+ 6x, then y = f[g(x)].... 8373+01274+93738939x937x058+938484 Determine if the set of number of days in January, J, and the set B={6,9,15,10,1,0,8,4,11} is equivalent Divide 12x5 36x4 6x3 by 3x2. 4x2 + 12x + 2 4x2 12x 2 4x3 + 12x2 + 2x 4x3 12x2 2x You have 4929 songs in your computers music library. The songs have a mean duration of 246.3 seconds with a standard deviation of 111.42 seconds. One of the songs is 391 seconds long. What is its z-score? Match each graph with the correct equation from the equation bank not all equations will be used. (pls help) questions 6,7,8,10.